0

consider the Lagrangian 1$/$2 m$\dot{x}$$^{2}$ $-$ V$($x$)$ such that x $=$ f$($t$)$ , $\dot{x}$ $=$ g$($t$)$ and t $=$ f$^{-1}$$($x$)$ , $\dot{x}$ $=$ g$($f$^{-1}$$($x$)$$)$ and, $\frac{\partial{L}}{\partial{x}}$ $=$ $\frac{d}{dt}$ $($ $\frac{\partial{L}}{\partial{\dot{x}}}$ $)$ . Now we know that $\frac{\partial{L}}{\partial{\dot{x}}}$ $=$ m$\dot{x}$ ...$($1$)$.But I can say that $\frac{\partial{L}}{\partial{\dot{x}}}$ $=$ m$\dot{x}$ $\frac{\partial{\dot{x}}}{\partial{x}}$.......$($ 2$)$.

According to me $\frac{\partial{\dot{x}}}{\partial{x}}$ is zero

Because $\partial{\dot{x}}$ is 0 since by the definition of partial differentiation we keep t constant that is t doesn't change ! That is why the above quantity is 0 and we reduce back to 1) from 2) . Now all that I want to know is that whether this is right or not ?

This question is an extension of Why does $\frac{dq}{dt}$ not depend on $q$? Why does the calculus of variations work?

Shashaank
  • 933
  • (2) is wrong. Where in does the derivative with respect to $x$ come from? – Dr. Wolfgang Hintze Jun 06 '17 at 11:24
  • @Dr.WolfgangHintze because I have shown that $\dot{x}$ depends on x i.e velocity depends on displacement !! – Shashaank Jun 06 '17 at 11:35
  • There are two misconceptions in your answer: 1) as conjugate variables $p ( =m \dot x)$ and $x$ are idependent 2) formally, you differentiate with respect to $\dot x$, and there is no reason for a derivative with respect to $x$ to appear (make a dimension analysis to falsify your formula). – Dr. Wolfgang Hintze Jun 06 '17 at 12:15
  • Possible duplicates: https://math.stackexchange.com/q/580858/11127 and links therein. – Qmechanic Jun 06 '17 at 13:20
  • @Dr.WolfgangHintze I couldn't understand ...... Because x dot depends on x....right....the link which I have given , people have agreed with that ( in the comments and the answer).... So I should apply chain rule... – Shashaank Jun 06 '17 at 15:56
  • @Shashaank Please have a look at this link you gave inderectly yourself https://physics.stackexchange.com/questions/885/why-does-calculus-of-variations-work/2908#2908. Here Greg Graviton said what is necessary, and then, using this input, grizzly adam who put the question answered it himself. – Dr. Wolfgang Hintze Jun 07 '17 at 19:46
  • @Dr.WolfgangHintze Sorry , I might be wrong but I think they are wrong. It would be helpful if you could explain where I am wrong .Have a look at this " Position and velocity are independent. They depend explicitly only on time. They off course depend implicitly on each other but not at all explicitly. You can not change v by just changing x alone. When you change x it is understood that t changes. It is because of that change in t that v changes. Essentially the partial derivative of v with x is 0 but the derivative of v with x is not 0. That is why, I think we don't apply any " chain rule " – Shashaank Jul 31 '17 at 19:17

0 Answers0